Jump to content

New simulation shows Einstein was correct about hidden variables


Theoretical

Recommended Posts

My example demonstrated that the polarization can not be determined when the photon leaves the source. If it was determined then the angle of Bob's detector should not affect Alice's results. If it leaves a 1 then Alice should get a 1 no matter what the angle of Bob's detector is. My example showed that if Alice's detector is at 30 degrees then her results with bob at 0 would be different then her results with bob at -30 degrees. The polarization can not be determined when it leaves the source. The polarization is random until it is measured. It doesn't have an exact polarization until it is measured. It should be noted that this can not be used for faster then light communication. Alice's signal will appear as a random string of zeroes and ones no matter what Bob's angle is. The only way they can determine entanglement occurred is if they come together and compare the results. This would involve communication at the speed of light or slower.

Link to comment
Share on other sites

My example demonstrated that the polarization can not be determined when the photon leaves the source. If it was determined then the angle of Bob's detector should not affect Alice's results. If it leaves a 1 then Alice should get a 1 no matter what the angle of Bob's detector is. My example showed that if Alice's detector is at 30 degrees then her results with bob at 0 would be different then her results with bob at -30 degrees. The polarization can not be determined when it leaves the source. The polarization is random until it is measured. It doesn't have an exact polarization until it is measured. It should be noted that this can not be used for faster then light communication. Alice's signal will appear as a random string of zeroes and ones no matter what Bob's angle is. The only way they can determine entanglement occurred is if they come together and compare the results. This would involve communication at the speed of light or slower.

Sure it can. Just put a polarizer right after the photon emitter. That's what I was saying. Bell's experiment using non-entangled photons gets the same result as Bell's experiment with entangled photons.
Link to comment
Share on other sites

Sure it can. Just put a polarizer right after the photon emitter. That's what I was saying. Bell's experiment using non-entangled photons gets the same result as Bell's experiment with entangled photons.

Actually that would not work unless the polarizer is literally touching the source. If there is any distance between the polarizer and the source then It's polarization is undetermined until it is measured. The experiment does not get the same result if the sources are not entangled. Bob's signal would not match Alice's at 0 degrees if the signals are not entangled. I have clearly demonstrated an example where the polarization can not be determined when it leaves the source. You respond by saying we should change the experiment. Why don't we just change the experiment to you jumping up and down. That experiment would show no signs of entanglement.
Link to comment
Share on other sites

Actually that would not work unless the polarizer is literally touching the source. If there is any distance between the polarizer and the source then It's polarization is undetermined until it is measured. The experiment does not get the same result if the sources are not entangled. Bob's signal would not match Alice's at 0 degrees if the signals are not entangled. I have clearly demonstrated an example where the polarization can not be determined when it leaves the source. You respond by saying we should change the experiment. Why don't we just change the experiment to you jumping up and down. That experiment would show no signs of entanglement.

Photons don't care when they go through the polarizers. Yes it does get the same results. My sim shows that.
Link to comment
Share on other sites

Photons don't care when they go through the polarizers. Yes it does get the same results. My sim shows that.

How would Bob and Alice's signal match at 0 degrees if they are not entangled? They would get two different random strings of zeros and ones. There are many things that I can do in video games which can not be done in real life. This is why sims don't prove real life. Edited by david345
Link to comment
Share on other sites

How would Bob and Alice's signal match at 0 degrees if they are not entangled? They would get two different random strings of zeros and ones. There are many things that I can do in video games which can not be done in real life. This is why Sims don't prove real life.

Ah, so you're saying QM doesn't work because entering data on a calculator is not real life.

You're not correctly doing a Bell's experiment if you think the entangled photons are always the same polarity. And yes, the setup I described can make both non-entangled photons any polarity they want.

Link to comment
Share on other sites

Ah, so you're saying QM doesn't work because entering data on a calculator is not real life.

You're not correctly doing a Bell's experiment if you think the entangled photons are always the same polarity. And yes, the setup I described can make both non-entangled photons any polarity they want.

According to you Bell's experiment is only done correctly if it demonstrates Bell's theory to be wrong. The experiment uses a beam splitter which ensures photons are sent out with matching polarazition.
Link to comment
Share on other sites

I'm only saying Bell's experiment gives same results for entangled or non-entangled photons so long as the setup is such that photon polarization are the same, which is not difficult, and does not prove spooky action at a distance. That's all.

Link to comment
Share on other sites

I'm only saying Bell's experiment gives same results for entangled or non-entangled photons so long as the setup is such that photon polarization are the same, which is not difficult, and does not prove spooky action at a distance. That's all.

Actually it does prove spooky action at a distance. My example demonstrated the angle of Bob's detector affects Alice's results. If the polarization was determined when it left the source then the angle of Bob's detector should not effect Alice's results. If it left as 1 then Alice should detect a 1 no matter what Bob's angle is. My example demonstrated that is not the case.
Link to comment
Share on other sites

Actually it does prove spooky action at a distance. My example demonstrated the angle of Bob's detector affects Alice's results. If the polarization was determined when it left the source then the angle of Bob's detector should not effect Alice's results. If it left as 1 then Alice should detect a 1 no matter what Bob's angle is. My example demonstrated that is not the case.

You miss that the experiment gets the same results with non-entangled photons that have the same polarities as well.
Link to comment
Share on other sites

You miss that the experiment gets the same results with non-entangled photons that have the same polarities as well.

I demonstrated an experiment where the polarization can not be determined until it is measured. If it was determined before measurement then the mismatch would be 50% and not 75%. Let us assume you are correct. Your non entangled photons with matching polarization get the same 75% mismatch percentage. These "non entangled" photons would still not have an exact polarization until they are measured. You would still need action at a distance to explain the result. The angle of Bob's detector will still affect what Alice detects. If they had a exact polarization when they left the source then you would only get a 50% mismatch. Edited by david345
Link to comment
Share on other sites

Placing a polariser at the source would break any entanglement and superposition in the polarisation. Your results would then be meaningless.

 

Also photons do "care" if you put polarisers in the way, even classically that's why you can block light with two polarisers but introducing a third between the first two allows light to pass through.

 

Theoretical, if you're really interested in this, and you seem to be, I strongly suggest you go away and try and learn some more on the topic in a note formal way then videos. Videos are good for wetting the appetite but to really understand this you need to study for a few years learning everything from the basics up. This is probably why you are not getting why people are arguing that your sim isn't showing what you think it is.

Link to comment
Share on other sites

No need for you people to insult me just because I don't agree with you. Those kind of comments are just personal and insulting. It's okay if people don't agree. Let other people make up their own mind.

 

1. You can't tell me I can't send two photons at the same time from two different sources that according to QM are not entangled photons. One could even do this with radio wavelength photons. I've created thousands of antenna experiments using the world known NEC (Numerical Electromagnetics Code) radio wave engine created by Lawrence Livermore National Laboratory. Cool two horizontally polarized antennas. A vertical antenna does not pick up horizontally polarized photons, but yet a horizontal antenna will. Furthermore it's extremely simple to build a radio polarizer. An array of horizontal dipoles reflects horizontal photons, while allowing vertical photons to pass through.

 

2. You can't tell me the results of such non-entangled photons in a Bell experiment setup does not results in the same polarization hits half of the time, which is what QM predicts for entangled photons. You can't tell me that because I've go through this by showing on paper the path each photon takes using the well known cos(angle)^2 equation discovered in the 1700's. I could create a video that clearly shows the path of each photon, counting each 0 and 1. The video would show this results in 1/2.

 

Honestly I'm done trying to convince people of the obvious. It's not my fault there's no evidence of spooky action at a distance. I get it that a lot of people are heavily invested in this spooky action at a distance notion. It makes a lot of money for research scienctists. The medium eats it up alive.

 

If you want to see this sim for yourself, then please by all means get out a piece of paper, *literally* track the paths of each photon using the cos(angle)^2 equation, count all the times both photons take the same path. See for yourself.

 

 

Also photons do "care" if you put polarisers in the way,

Sorry to have to point this out, but the above is an example of how my words are consistently being twisted. I never said the above. I clearly did not say that. Here's my quote:

 

Photons don't care ***when*** they go through the polarizers.

I get it that this spooky action at a distance rings in mega bucks for QM.

 

 

Ok I'm going back to my research. :) Please by all means feel free to misquote me lol. Peace, and hopefully will see you all soon.

Link to comment
Share on other sites

No need for you people to insult me just because I don't agree with you. Those kind of comments are just personal and insulting. It's okay if people don't agree. Let other people make up their own mind.

 

I see no insults. But if you think there are, just use the Report link at the bottom of the offending post.

 

Honestly I'm done trying to convince people of the obvious.

 

The obviously wrong. I regret giving you a +1 when it seemed you had finally agreed to go and learn what Bell's theorem really says, rather than your meaningless version.

 

It's not my fault there's no evidence of spooky action at a distance.

 

Why do you keep repeating this lie?

 

There is masses of evidence. It is even being developed as a technology in quantum computing and secure communication.

 

I get it that a lot of people are heavily invested in this spooky action at a distance notion. It makes a lot of money for research scienctists.

 

Scientists would love to prove it wrong - it would guarantee them a Nobel Prize.

 

Ok I'm going back to my research.

 

Good. I hope this means studying (i.e. READING not watching youtoob videos) and finding out what Bell's theorem is really about.

Link to comment
Share on other sites

If you want to see this sim for yourself, then please by all means get out a piece of paper, *literally* track the paths of each photon using the cos(angle)^2 equation, count all the times both photons take the same path. See for yourself.

 

post-107966-0-92349400-1420407573_thumb.jpgpost-107966-0-52205700-1420407584_thumb.jpg

We will assume 4 "unentangled" photons with matching polarizations are sent out in both directions. Their polarization will be determined when they leave the source. The detectors will read a 0 or 1 depending on the axis of polarization. We will set the polarizations to 1010 for the 4 photons.

 

step 1: Bob and Alice set their detectors at matching angles. They both get 1010.

 

step 2: The same 1010 signal is sent out. Alice rotates detector 30 degrees. The cos(angle)^2 equation says she should get a 25% error rate. Bob gets 1010 Alice gets 1110. A 25% mismatch.

 

step 3: The same 1010 signal is sent out. Alice returns detector to original position and now Bob rotates his detector -30%. The cos(angle)^2 equation says Bob should get a 25% error rate. Bob gets 1000 and Alice gets 1010. A 25% mismatch.

 

Step 4: The same 1010 signal is sent out. Alice rotates her detector 30 degrees. Bob rotates his detector -30 degrees. Alice should get 1110 the same result as step 2. The exact same signal was sent out. Her detector was in the exact same 30 degree position. She should get the exact same result. Bob should also get the exact same 1000 result as step 3. It was the exact same 1010 signal sent out and his detector is in the exact same -30 degree position. Let us compare results. Alice 1110 Bob 1000. This is a 50% mismatch. The cos(angle)^2 equation says there should be a 75% mismatch. The only way you can get this 75% mismatch is if the polarizations are not determined until they are measured and the angle of Bob's detector affected Alice's results and the angle of Alice's detector affected Bob's result. A 75% mismatch can only be explained by spooky action at a distance. You claim you "researched" Bell's theory in hope of finding an example of spooky action at a distance. Here it is.

Edited by david345
Link to comment
Share on other sites

EPR1J.jpg EPR2J.jpg

We will assume 4 "unentangled" photons with matching polarizations are sent out in both directions. Their polarization will be determined when they leave the source. We will set the polarization to 1010.

 

step 1: Bob and Alice set their detectors at matching angles. They both get 1010.

 

step 2: The same 1010 signal is sent out. Alice rotates detector 30 degrees. The cos(angle)^2 equation says she should get a 25% error rate. Bob gets 1010 Alice gets 1110. A 25% mismatch.

 

step 3: The same 1010 signal is sent out. Alice returns detector to original position and now Bob rotates his detector -30%. The cos(angle)^2 equation says Bob should get a 25% error rate. Bob gets 1000 and Alice gets 1010. A 25% mismatch.

 

Step 4: The same 1010 signal is sent out. Alice rotates her detector 30 degrees. Bob rotates his detector -30 degrees. Alice should get 1110 the same result as step 2. The exact same signal was sent out. Her detector was in the exact same 30 degree position. She should get the exact same result. Bob should also get the exact same 1000 result as step 3. It was the exact same 1010 signal and his detector is in the exact same -30 degree position. Let us compare results. Alice 1110 Bob 1000. This is a 50% mismatch. The cos(angle)^2 equation says there should be a 75% mismatch. The only way you can get this 75% mismatch is if the polarizations are not determined until they are measured and the angle of Bob's detector affected Alice's results and the angle of Alice's detector affected Bob's result. A 75% mismatch can only be explained by spooky action at a distance. You claim you "researched" Bell's theory in hope of finding an example of spooky action at a distance. Here it is.

Please do yourself a favor and look at your drawings to see how they show no details and consist of assumptions.

 

What I would suggest is that you go all out like I did and track every single photon to each of their polarizer, use the only equation that works in real life for this: cos(angle)^2, do this for at least a few hundred photons, compute the average percentage of time the photons are go to the same path (that includes photons that reflect off the polarizer as well), and verify that non-entangled photons also result in 50%.

 

 

If you keep this up then eventually I'm going to produce a clear cut video of my sim, which clearly shows non-entangled photon Bell's experiment results in 1/2. And I'll also include details of NEC radio wavelength experiment which shows the same results. So look out my friend lol. ;-)

Link to comment
Share on other sites

If you keep this up then eventually I'm going to produce a clear cut video of my sim, which clearly shows non-entangled photon Bell's experiment results in 1/2. And I'll also include details of NEC radio wavelength experiment which shows the same results. So look out my friend lol. ;-)

I will be waiting.
Link to comment
Share on other sites

Thank you. I love to see a video you create as well. Just please do me a favor: while creating your video, go over every single statement you make and show, making sure there are no assumptions because remember people aren't mind readers. Also a lot of non-technical people view these videos.

Link to comment
Share on other sites

Please do yourself a favor and look at your drawings to see how they show no details and consist of assumptions.

What I would suggest is that you go all out like I did and track every single photon to each of their polarizer, use the only equation that works in real life for this: cos(angle)^2, do this for at least a few hundred photons, compute the average percentage of time the photons are go to the same path

As I clearly demonstrated the photons can not have a polarization until they are measured. It is nonsensical to track their polarization all the way to the detector. You can not track what does not exist. The path they take is the dotted line. The 4 photons are released one after another. As I stated earlier more photons would mean more zeros and ones. The average percentage is given by the cos(angle)^2 equation. It is your code which does not do anything more then spit out the number 1\2. This is why you have not once demonstrated your code in action. Your code does not work. It is worthless and this is why you have provided no demonstration. You can even post a video of your code running on YouTube. This would allow us to watch it track each photon. I expect you to come back with something better then "It works because I say so". I expect your code to do something more then spit out the number 1\2. Edited by david345
Link to comment
Share on other sites

As I clearly demonstrated the photons can not have a polarization until they are measured.

You couldn't possibly expect any physicist to believe that. Emitting polarized photon is extremely easy even in radio wavelenthgs. Nearly every antenna emits polarized photons, and that is a fact.

 

And who cares if it's been measured? You have a black box that emits the photons.

Link to comment
Share on other sites

 

As stated, my sim is all about emitting photons where the polarization is known beforehand. It makes absolutely no difference if the photon polarity is guaranteed the moment the antenna or whatever emitted it or if we want to put it through a polarizer.

 

Everyone here knows we can produce photons of a specific polarization.

Link to comment
Share on other sites

As stated, my sim is all about emitting photons where the polarization is known beforehand. It makes absolutely no difference if the photon polarity is guaranteed the moment the antenna or whatever emitted it or if we want to put it through a polarizer.

Everyone here knows we can produce photons of a specific polarization.

Your code does not work. This is why you can provide no demonstration. Just excuses.
Link to comment
Share on other sites

 

 

Everyone except you.

 

Still waiting for that demonstration.

You need to learn physics 101:

 

http://en.m.wikipedia.org/wiki/Polarizer

"A polarizer or polariser is an optical filter that passes light of a specific polarization and blocks waves of other polarizations."

Your code does not work. This is why you can provide no demonstration. Just excuses.

I provided source code. And someone in this thread created a math equation of it. :)
Link to comment
Share on other sites

You need to learn physics 101:http://en.m.wikipedia.org/wiki/Polarizer

"A polarizer or polariser is an optical filter that passes light of a specific polarization and blocks waves of other polarizations."

More excuses. Your code does nothing more then spit out the number 1\2. http://en.m.wikipedia.org/wiki/Bell%27s_theorem Edited by david345
Link to comment
Share on other sites

Guest
This topic is now closed to further replies.
×
×
  • Create New...

Important Information

We have placed cookies on your device to help make this website better. You can adjust your cookie settings, otherwise we'll assume you're okay to continue.